Tag Archives: russell hibbeler

Hibbeler Statics 14E P2.4 – Components of a Force Along Two Non-Perpendicular Axes


The vertical force \textbf{F} acts downward at A on the two-membered frame. Determine the magnitudes of the two components of \textbf{F} directed along the axes of AB and AC. Set \textbf{F} = 500 N.

Engineering Mechanics: Statics 14th Edition by RC Hibbeler, Problem 2-4


Solution:

Draw the components of the force using the parallelogram law. Then the triangulation rule.

Parallelogram Law
Triangulation Rule

Solving for FAC using sine law.

\begin{align*}
\frac{\text{F}_\text{AC}}{\sin \ 45^\circ } & =\frac{500 \ \text{N}}{\sin \ 75^\circ }\\
\text{F}_\text{AC} & = \frac{500 \ \text{N} \ \sin45^\circ }{\sin\ 75^\circ }\\
\text{F}_\text{AC} & =366.0254 \ \text{N}\\
\text{F}_\text{AC} &\approx 366 \ \text{N}
\end{align*}

Solve for FAB using sine law.

\begin{align*}
\frac{\text{F}_\text{AB}}{\sin \ 60^\circ } & =\frac{500 \ \text{N}}{\sin \ 75^\circ }\\
\text{F}_\text{AB} & = \frac{500 \ \text{N} \ \sin60^\circ }{\sin\ 75^\circ }\\
\text{F}_\text{AB} & =448.2877 \ \text{N}\\
\text{F}_\text{AB} &\approx 448 \ \text{N}
\end{align*}

Advertisements
Advertisements

Hibbeler Statics 14E P2.2 — Solving for an Unknown Force Given the Magnitude and Direction of a Resultant and Another Force


If the magnitude of the resultant force is to be 500 N, directed along the positive y-axis, determine the magnitude of force F and its direction \theta .

Engineering Mechanics: Statics 14th Edition by RC Hibbeler, Problem 2-2


Solution:

The parallelogram law and the triangulation rule are shown in the figures below.

Engineering Mechanics by RC Hibbeler Problem 2.2 Parallelogram Law
Parallelogram Law
Engineering Mechanics by RC Hibbeler Problem 2.2 Triangulation Rule
Triangulation Rule

Considering the figure of the triangulation rule, we can solve for the magnitude of \textbf{F} using the cosine law.

\begin{align*}
\textbf{F} & = \sqrt{700^2+500^2-2\left( 700 \right)\left( 500 \right)\cos105^{\circ}}\\
& = 959.78 \  \text{N}\\
& = 960 \  \text{N}\\
\end{align*}

Then we use the sine law to solve for the angle \theta.

\begin{align*}
\frac{\sin \left(90^{\circ}-\theta \right)}{700} & = \frac{\sin 105^{\circ}}{959.78}\\
\sin \left(90^{\circ}-\theta \right) & =\frac{700 \sin 105^{\circ }}{959.78}\\
90^{\circ}-\theta & = \sin^{-1} \left( \frac{700 \sin 105^{\circ }}{959.78} \right)\\
\theta & = 90^\circ-\sin^{-1} \left( \frac{700 \sin 105^{\circ }}{959.78} \right) \\
\theta & =  90^\circ-44.79^\circ\\
\theta & =  45.2^\circ\\
\end{align*}

Advertisements
Advertisements

Hibbeler Statics 14E P2.1 — Solving for the Magnitude and Direction of the Resultant of Two Coplanar-Concurrent Forces


If \theta = 60 \degree and \textbf{F} = 450 \ \text{N}, determine the magnitude of the resultant force and its direction, measured counterclockwise from the positive x axis.

Engineering Mechanics: Statics 14th Edition by RC Hibbeler, Problem 2-1


Solution:

The parallelogram law and the triangulation rule are shown in the figures below.

(a) Parallelogram Law
(b) Triangulation Rule

Considering figure (b), we can solve for the magnitude of \textbf{F}_R using the cosine law.

\begin{align*}
\textbf{F}_R & = \sqrt{700^2+450^2-2\left( 700 \right)\left( 450 \right)\cos45^{\circ}}\\
& = 497.01 \ \text{N}\\
& = 497 \ \text{N}
\end{align*}

Then we use the sine law to solve for the interior angle \theta.

\begin{align*}
\frac{\sin \theta}{700} & = \frac{\sin 45^{\circ}}{497.01}\\
\sin \theta & =\frac{700\ \sin 45^{\circ }}{497.01}\\
\theta & = \sin^{-1} \left( \frac{700\ \sin 45^{\circ }}{497.01} \right)\\
& \text{This is an ambiguous case }\\
\theta & = 84.81^\circ \  or \  \theta =95.19^\circ \\
\end{align*}

In here, the correct angle measurement is \theta = 95.19^{\circ}.

Thus, the direction angle \phi of \textbf{F}_R measured counterclockwise from the positive x-axis, is

\begin{align*}
\phi & = \theta +60^\circ \\
& = 95.19^\circ +60^\circ \\
& = 155^\circ 
\end{align*}

Advertisements
Advertisements

Hibbeler Statics 14E P1.19 — Determine the Weight of the Column with a given Density


A concrete column has a diameter of 350 mm and a length of 2 m. If the density (mass/volume) of concrete is 2.45 Mg/m3, determine the weight of the column in pounds.

Engineering Mechanics: Statics 14th Edition by RC Hibbeler, Problem 1-19


Solution:

The density of any material is given by the formula

\text{density}=\frac{\text{mass}}{\text{volume}}

From there, we can compute for the mass as

\text{mass}=\text{density} \times \text{volume}

We can solve for mass by multiplying density by volume. The density is already given, and we can compute for the volume of the concrete column by the formula of a volume of a cylinder.

\begin{align*}
\text{V} & = \pi \text{r}^2 \text{h}\\
& =\pi \left( \frac{0.35\ \text{m}}{2} \right)^2 \left( 2 \ \text{m} \right)\\
& =0.1924 \ \text{m}^3
\end{align*}
Concrete Column illustration with diameter of 350 mm or 0.35 m, and a height of 2 m

Therefore, the mass of the concrete column is

\begin{align*}
\text{mass} & =\text{density} \times \text{volume}\\
& = \left( 2.45 \times 10^3 \ \text{kg/m}^3 \right)\times \left( 0.1924 \ \text{m}^3 \right)\\
& =471.44 \ \text{kg}\\
\end{align*}

Now, we can solve for the weight by multiplying the mass by the acceleration due to gravity, g.

\begin{align*}
\text{Weight} & = \text{mass} \times \text{acceleration due to gravity} \\
& = 471.44 \ \text{kg} \times 9.81 \ \text{m/s}^2 \\
& = 4624.78 \ \text{N}
\end{align*}

Finally, we can convert the weight in Newtons to weight in pounds.

\begin{align*}
4624.78\ \text{N} & = 4624.78\ \text{N}\times \frac{1\ \text{lb}}{4.4482\ \text{N}}\\
& = 1039.70\ \text{lb}\\
& = 1.04\times 10^3 \  \text{lb}\\
& = 1.04 \ \text{kip}
\end{align*}

Advertisements
Advertisements

Hibbeler Statics 14E P1.11 — Representing Measurements with SI units Having Appropriate Prefix


Represent each of the following with SI units having an appropriate prefix: (a) 8653 ms, (b) 8368 N, (c) 0.893 kg.

Statics of Rigid Bodies 14th Edition by RC Hibbeler, Problem 1-11


Solution:

Part A

\begin{align*}
8653 \ \text{ms} & = 8653 \ \left( 10^{-3} \right) \ \text{s} \\
& =8.653 \ \text{s}
\end{align*}

Part B

\begin{align*}
8368 \ \text{N} & = 8.368\times 10^3 \ \text{N}\\
& = 8.368 \ \text{kN}\\
\end{align*}

Part C

\begin{align*}
0.893 \  \text{kg} & = 0.893\times 10^3 \ \text{g} \\
& =893 \ \text{g}
\end{align*}

Advertisements

Hibbeler Statics 14E P1.10 — Representing Combinations of Units in the Correct SI Form


Represent each of the following combinations of units in the correct SI form: (a) GNµm, (b) kg/µm, (c) N/ks2, and (d) KN/µs.

Statics of Rigid Bodies 14th Edition by RC Hibbeler, Problem 1-10


Solution:

Part A

\begin{align*}
\text{GN} \cdot  \mu \text{m} & = \left( 10^9 \ \text{N} \right)\left( 10^{-6} \ \text{m} \right)\\
& = 10^3 \ \text{N} \cdot \text{m}\\
& = \text{kN} \cdot \text{m}
\end{align*}

Part B

\begin{align*}
\text{kg/}\mu\text{m} & = \frac{10^3 \ \text{g}}{10^{-6} \ \text{m}} \\
& = 10^9 \ \frac{\text{g}}{\text{m}} \\
& = \text{Gg/m}
\end{align*}

Part C

\begin{align*}
\text{N/ks}^2 & = \frac{\text{N}}{\left( 10^3 \ \text{s} \right)^2}\\
& = \frac{\text{N}}{10^6 \ \text{s}^2} \\
& = 10^{-6} \ \frac{\text{N}}{\text{s}^2} \\
& = \mu \text{N}/\text{s}^2
\end{align*}

Part D

\begin{align*}
\text{kN}/ \mu\text{s} & = \frac{10^3 \ \text{N}}{10^{-6} \ \text{s}} \\
& = 10^9 \ \frac{\text{N}}{\text{s}}\\
& = \text{GN/s}
\end{align*}

Advertisements
Advertisements

Statics 3.3 – Solving for the magnitude and direction of a force for equilibrium | Hibbeler 14th Edition


Determine the magnitude and direction θ of F so that the particle is in equilibrium.

Statics 14E Problem 3.3 Forces in Equilibrium with unknown force and its direction

Solution:

Free-body Diagram:

Equilibrium Equation:

Summation of forces in the x-direction:

\begin{aligned}
\xrightarrow{+} \: \sum F_x & = 0 & \\
5 \: \text{kN}+F \sin \theta - 8 \: \text {kN} \cos 30 \degree - 4\: \text{kN} \cos 60 \degree & = 0  & \\
F \sin \theta &= 3.9282  & (1)

\end{aligned}

Summation of forces in the y-direction:

\begin{aligned}
+\uparrow \sum F_y & = 0  &\\
8 \sin 30 \degree - 4 \sin 60 \degree - F \cos \theta & =0 &\\
F \cos \theta & = 0.5359 & (2)\\

\end{aligned}

We now have two equations. Divide Eq (1) by (2)

\begin{aligned}
\dfrac{F \sin \theta}{F \cos \theta}  &= \dfrac{3.9282}{0.5359} \\

\dfrac{ \sin \theta}{ \cos \theta} & = 7.3301 \\ 

\end{aligned}

We know that \tan \theta = \dfrac{\sin \theta}{\cos \theta} :

\begin{aligned}
\tan \theta &=7.3301 \\
\theta & = \tan^{-1}7.3301\\
\textcolor{blue}\theta & \textcolor{blue}{=82.2\degree}\\
\end{aligned}

Substituting this result to equation (1), we have

\begin{aligned}
F\sin 82.2 \degree & = 3.9282 \\
\textcolor{blue}F & \textcolor{blue}{=3.96 \ \text{kN}}
\end{aligned}

Statics 3.2 – Equilibrium of Truss Members that are Pin Connected | Hibbeler 14th Edition


The members of a truss are pin connected at joint O. Determine the magnitude of F1 and its angle θ for equilibrium. Set F2=6 kN.

Figure 3.1: Engineering Mechanics: Statics Equilibrium of Particle
Figure 3.1/3.2

Solution:

Free-body diagram:

Free-body-diagram-for-Problem-3.2 of Engineering Mechanics: Statics by Russell C. Hibbeler

Equations of Equilibrium:

The summation of forces in the x-direction:

\begin{aligned}
\sum F_x & = 0 &\\
6 \sin 70 \degree + F_1 \cos \theta - 5 \cos 30 \degree - \dfrac {4}{5} \left(7 \right) & = 0 & \\
 F_1 \cos \theta & = 4.2920 & (1)
\end{aligned}

The summation of forces in the y-direction:

\begin{aligned}
\sum F_y & =0 & \\
6 \cos 70 \degree+5 \sin 30 \degree - F_1 \sin \theta - \dfrac{3}{5} \left( 7 \right ) & =0 & \\
F_1 \sin \theta &=0.3521 & (2)\\
\end{aligned}

We came up with 2 equations with unknowns F_1 and \theta . To solve the equations simultaneously, we can use the method of substitution.

Using equation 1, solve for F_1 in terms of \theta .

\begin{aligned}
F_1 \cos \theta & = 4.2920  &\\
F_1 & =\dfrac{4.2920}{ \cos \theta } & (3) \\
\end{aligned}

Now, substitute this equation (3) to equation (2).

\begin{aligned}
F_1 \sin \theta & = 0.3521 \\
\left ( \dfrac {4.2920}{\cos \theta} \right) \sin \theta & =0.3521 \\
4.2920 \cdot \dfrac{\sin \theta}{\cos \theta} & = 0.3521 \\
4.2920 \tan \theta & = 0.3521 \\
\tan \theta & = \dfrac{0.3521}{4.2920} \\
\theta &= \tan ^{-1} \dfrac{0.3521}{4.2920} \\
\theta & = 4.69 \degree

\end{aligned}

Substitute the solved value of \theta to equation (3).

\begin{aligned}
F_1 & = \dfrac{4.2920}{\cos \theta} \\
F_1 &= \dfrac{4.2920}{\cos 4.69 \degree} \\
F_1 & = 4.31 \text{kN}
\end{aligned}

Therefore, the answers to the questions are:

\begin{aligned}
F_1= & \:4.31 \: \text {kN} \\
\theta = & \: 4.69 \degree
\end{aligned} 

Statics 3.1 – Equilibrium of Truss Members in Pin Connection | Hibbeler 14th Edition


The members of a truss are pin connected at joint O. Determine the magnitudes of F1 and F2 for equilibrium. Set θ=60.

Figure 3.1: Engineering Mechanics: Statics Equilibrium of Particle
Figure 3.1

Solution:

Free-body diagram:

Free-Body Diagram for Problem 3.1 of Engineering Mechanics: Statics 14th Edition by Russell C. Hibbeler

Equations of Equilibrium:

Take the sum of horizontal forces considering forces to the right positive, and equate to zero.

\begin {aligned}


\sum{F}_x &= 0 & \\

F_1 \cos{60 \degree}+F_2 \sin{70 \degree}-5\cos{30 \degree}-\dfrac{4}{5}\left(7\right) &= 0 &\\

0.5F_1+0.9397F_2&=9.9301 &(1)\\

\end {aligned}

Take the sum of vertical forces considering upward forces positive, and equate to zero.

\begin{aligned}

\sum F_y&=0 &\\

-F_1\sin60\degree+F_2\cos70\degree+5\sin30\degree-\dfrac{3}{5}\left(7\right)&=0 &\\

-0.8660F_1+0.3420F_2&=1.7 &(2)\\

\end{aligned}

Now, we have two equations with two unknowns F_1 and F_2 . So, we have a system of two equations. We can solve this using algebra, or we can directly use our calculator with this capability. The answers are

F_1=1.83 \: \text{kN}\\
F_2=9.60 \: \text{kN}

Chapter 3: Equilibrium of a Particle

Coplanar Force Systems

Problem 3

Problem 4

Problem 5

Problem 6

Problem 7

Problem 8

Problem 9

Problem 10

Problem 11

Problem 12

Problem 13

Problem 14

Problem 15

Problem 16

Problem 17

Problem 18

Problem 19

Problem 20

Problem 21

Problem 22

Problem 23

Problem 24

Problem 25

Problem 26

Problem 27

Problem 28

Problem 29

Problem 30

Problem 31

Problem 32

Problem 33

Problem 34

Problem 35

Problem 36

Problem 37

Problem 38

Problem 39

Problem 40

Problem 41

Problem 42

Three-Dimensional Force Systems

Problem 43

Problem 44

Problem 45

Problem 46

Problem 47

Problem 48

Problem 49

Problem 50

Problem 51

Problem 52

Problem 53

Problem 54

Problem 55

Problem 56

Problem 57

Problem 58

Problem 59

Problem 60

Problem 61

Problem 62

Problem 63

Problem 64

Problem 65

Problem 66

Problem 67